Difference between revisions of "2023 AMC 12B Problems/Problem 7"
(Created blank page) |
|||
Line 1: | Line 1: | ||
+ | ==Solution== | ||
+ | We have | ||
+ | <cmath> | ||
+ | \begin{align*} | ||
+ | \sqrt{\frac{\log \left( n^2 \right) - \left( \log n \right)^2}{\log n - 3}} | ||
+ | & = \sqrt{\frac{2 \log n - \left( \log n \right)^2}{\log n - 3}} \\ | ||
+ | & = \sqrt{\frac{\left( \log n \right) \left( 2 - \log n\right)}{\log n - 3}} . | ||
+ | \end{align*} | ||
+ | </cmath> | ||
+ | |||
+ | Because <math>n</math> is an integer and <math>\log n</math> is well defined, <math>n</math> must be a positive integer. | ||
+ | |||
+ | Case 1: <math>n = 1</math> or <math>10^2</math>. | ||
+ | |||
+ | The above expression is 0. So these are valid solutions. | ||
+ | |||
+ | Case 2: <math>n \neq 1, 10^2</math>. | ||
+ | |||
+ | Thus, <math>\log n > 0</math> and <math>2 - \log n \neq 0</math>. | ||
+ | To make the above expression real, we must have <math>2 < \log n < 3</math>. | ||
+ | Thus, <math>100 < n < 1000</math>. | ||
+ | Thus, <math>101 \leq n \leq 999</math>. | ||
+ | Hence, the number of solutions in this case is 899. | ||
+ | |||
+ | Putting all cases together, the total number of solutions is | ||
+ | \boxed{\textbf{(E) 901}}. | ||
+ | |||
+ | ~Steven Chen (Professor Chen Education Palace, www.professorchenedu.com) |
Revision as of 17:27, 15 November 2023
Solution
We have
Because is an integer and is well defined, must be a positive integer.
Case 1: or .
The above expression is 0. So these are valid solutions.
Case 2: .
Thus, and . To make the above expression real, we must have . Thus, . Thus, . Hence, the number of solutions in this case is 899.
Putting all cases together, the total number of solutions is \boxed{\textbf{(E) 901}}.
~Steven Chen (Professor Chen Education Palace, www.professorchenedu.com)